20 votos

Ecuación geodésica a partir de la variación: ¿Es el lagrangiano al cuadrado equivalente?

Es bien sabido que las geodésicas en alguna variedad $M$ , cubierto por algunas coordenadas ${x_\mu}$ , digamos que con una métrica riemanniana se puede obtener por un principio de acción . Sea $C$ ser curva $\mathbb{R} \to M$ , $x^\mu(s)$ sea una parametrización afín de $C$ . (Aquí se utiliza el mismo símbolo para las coordenadas y la parametrización, pero es estándar). La acción que produce las geodésicas es:

$$ S (C) =\int_CL ds $$ donde $$ L\equiv \sqrt{g_{\mu\nu} \dot{x}^\mu \dot{x}^\nu} $$

Ahora el popular texto de Nakahara afirma que la variación de $$F \equiv \frac{L^2}{2}$$

Dará exactamente las mismas soluciones de principio de acción. Sin embargo, la referencia anterior SOLO muestra eso:

$C$ resuelve la ecuación de euler Lagrange para $L$ $\implies$ $C$ resuelve la ecuación de euler Lagrange para $F$

Y esto se puede demostrar mediante un cálculo directo de fuerza bruta.

Mi pregunta: ¿Es cierto lo contrario de lo anterior y cómo se puede demostrar? En concreto, ¿cómo demostramos que $F$ no produce soluciones extrañas a las ecuaciones de Euler Lagrange.

1 votos

Más sobre geodésicas a partir de principios variacionales: physics.stackexchange.com/q/67483/2451 , physics.stackexchange.com/q/94348/2451 , physics.stackexchange.com/q/137422/2451 y sus enlaces.

0 votos

Pregunta relacionada con Math.SE: math.stackexchange.com/q/1642719/11127

21voto

joshphysics Puntos 34367

El resultado de esta respuesta es el siguiente: si una trayectoria satisface las ecuaciones de Euler-Lagrange para $L^2/2$ entonces satisfará las ecuaciones de Euler-Lagrange para $L$ pero lo contrario no se cumple a menos que el camino tenga una parametrización afín.

Dejemos que $L = L(x, \dot x)$ sea un lagrangiano que sea una función local sólo de la posición y la velocidad, entonces una trayectoria parametrizada $x(s) = (x^i(s))$ en $M$ se dice que satisface las ecuaciones de Euler-Lagrange para $L$ proporcionado \begin{align} \frac{\partial L}{\partial x^i}(x(s), \dot x(s)) - \frac{d}{ds}\frac{\partial L}{\partial \dot x^i}(x(s), \dot x(s)) = 0 \end{align} para todos $i$ y para todos $s$ en el ámbito de $x$ .

Lema 1. Si $x$ satisface las ecuaciones de Euler-Lagrange para $L$ entonces la identidad de Beltrami es válida para $x$ :

$$ \frac{d}{ds}L(x(s), \dot x(s)) = \frac{d}{ds}\left(\frac{\partial L}{\partial \dot x^i}\big(x(s), \dot x(s)\big)\cdot \dot x^i(s)\right) $$

para todos $s$ en el ámbito de $x$ .

Prueba. Pruébelo usted mismo. La prueba se basa en el hecho de que $L$ es una función local de sólo $x$ y $\dot x$ .

Lema 2. Si $L(x,\dot x) = \sqrt{g_{ij}(x)\dot x^i\dot x^j}$ entonces $L$ satisface la siguiente identidad:

$$ \frac{\partial (L^2/2)}{\partial \dot x^i}(x, \dot x) \dot x^i = L(x,\dot x)^2 $$

Prueba. Pruebe usted también.

Corolario. Si $L(x,\dot x) = \sqrt{g_{ij}(x)\dot x^i\dot x^j}$ y $x$ satisface las ecuaciones de Euler-Lagrange para $L^2/2$ entonces $x$ satisface las ecuaciones de Euler-Lagrange para $L$ .

Prueba. Si $x$ satisface las ecuaciones de Euler-Lagrange para $L^2$ entonces el lema 1 da la siguiente identidad de Beltrami (utilizamos aquí una abreviatura notacional: todas las expresiones deben evaluarse en $x(s)$ )

$$ \frac{d(L^2/2)}{ds} = \frac{d}{ds} \frac{\partial (L^2/2)}{\partial \dot x^i}\cdot \dot x^i $$

Por otro lado, evaluando ambos lados del Lemma 2 en $x(s)$ y tomando la derivada de ambos lados con respecto a $s$ da

$$ \frac{d}{ds} \frac{\partial (L^2/2)}{\partial \dot x^i}\cdot \dot x^i = \frac{d(L^2)}{ds} $$

La combinación de estos hechos demuestra que $d(L^2)/ds = 0$ lo que implica que $L^2$ es constante a lo largo de $x(s)$ y por lo tanto que $L$ también es constante a lo largo de $x(s)$ :

$$ \frac{dL}{ds} = 0. $$

Ahora, observamos por separado que como $x$ satisface las ecuaciones de Euler-Lagrange para $L^2/2$ tenemos

\begin{align} 0 &= \frac{\partial(L^2/2)}{\partial x^i} - \frac{d}{ds} \frac{\partial (L^2/2)}{\partial \dot x^i} \\ &= L\left(\frac{\partial L}{\partial x^i} - \frac{d}{ds}\frac{\partial L}{\partial \dot x^i}\right) - \frac{dL}{ds}\frac{\partial L}{\partial \dot x^i} \tag{$\star$}\\ &= L\left(\frac{\partial L}{\partial x^i} - \frac{d}{ds}\frac{\partial L}{\partial \dot x^i}\right) \end{align}

y por lo tanto mientras $L\neq 0$ vemos que $x$ satisface las ecuaciones de Euler-Lagrange para $L$ como se deseaba.

El punto crucial aquí es que debido a la forma específica de $L$ cualquier trayectoria que satisfaga la ecuación de Euler-Lagrange para $L^2/2$ tiene la bonita propiedad de que $dL/ds = 0$ a lo largo del camino. Esto permite matar el término en $(\star)$ que es el término que constituye la diferencia esencial entre las ecuaciones de Euler-Lagrange para $L^2/2$ y las ecuaciones de Euler-Lagrange para $L$ .

Sin embargo, si $x$ satisface las ecuaciones de Euler-Lagrange para $L$ entonces no es necesariamente el caso que $dL/ds = 0$ a lo largo de $x$ , por lo que en este caso, no se puede matar ese término en $(\star)$ por lo que no es necesario que sea una solución de la ecuación de Euler-Lagrange para $L^2/2$ .

No obstante, si $x$ está parametrizado afinamente, entonces tendrá automáticamente la propiedad de que $L$ es constante a lo largo de ella, por lo que satisfará automáticamente las dos ecuaciones de Euler-Lagrange.

De hecho, utilizando partes de los cálculos anteriores, no es difícil demostrar que

Propuesta. Dejemos que $L(x, \dot x) = \sqrt{g_{ij}(x)\dot x^i\dot x^j}$ . Un camino $x$ es una geodésica afinamente parametrizada si y sólo si resuelve las ecuaciones de Euler-Lagrange de ambos $L$ y $L^2/2$ .

Así, las ecuaciones de Euler-Lagrange de $L^2/2$ dan lugar a todas las geodésicas con parámetros afines, mientras que las ecuaciones de Euler-Lagrange de $L$ producen todas las geodésicas, independientemente de la parametrización.

7voto

Stefano Puntos 763

Comentarios a la pregunta (v4):

  1. La formulación de la pregunta parece hablar de parametrizaciones afines antes de aplicando la principio de acción estacionaria .

  2. En el contexto de Riemann $^1$ geometría, un parametrización afín de una curva (no necesariamente geodésica) significa por definición que la longitud de arco $s$ y el parámetro de la curva $\lambda$ son afinadamente relacionado $s=a\lambda +b$ .

  3. Sin embargo, no siempre es posible mantener que todas las trayectorias virtuales [que satisfacen las condiciones de contorno pertinentes (BC)] estén parametrizadas afinamente con la el mismo parámetro común $\lambda$ . Obsérvese, en particular, que los BC se refieren al el mismo valores iniciales y finales $\lambda_i$ y $\lambda_f$ para todos los caminos.

  4. Por lo tanto, un a priori hay que abandonar el requisito de la parametrización afín de las trayectorias virtuales.

  5. También cualquier parcial a priori El requisito de parametrización afín sería ilógico, ya que imaginamos que no conocemos las geodésicas de antemano. De hecho, eso es lo que pretendemos encontrar con el principio de acción estacionaria.

  6. Definir el lagrangiano $$\tag{1}L_0(x,\dot{x})~:=~ g_{ij}(x) \dot{x}^i \dot{x}^j~\geq~0,$$ donde el punto significa la diferenciación con respecto al parámetro $\lambda$ .

  7. Ahora la acción de la raíz cuadrada $$\tag{2}S[x]~=~\int_{\lambda_i}^{\lambda_f}\! d\lambda \sqrt{L_0}$$ es invariable bajo reparametrizaciones de $\lambda$ por lo que las soluciones estacionarias para (2) con BCs pertinentes serán todas las geodésicas que satisfagan las BCs, independientemente de la parametrización.

  8. Por otro lado, las soluciones estacionarias para la acción de la raíz no cuadrada $$\tag{3}S_0[x]~=~\int_{\lambda_i}^{\lambda_f}\! d\lambda ~L_0$$ con las BCs pertinentes serán sólo todas las geodésicas parametrizadas afinamente que satisfagan las BCs.

  9. Más detalladamente, porque $L_0$ en la acción (3) no depende explícitamente de $\lambda$ se puede demostrar (por ejemplo, mediante el uso de Teorema de Noether ) que la función de energía $$\tag{4}h~:=~p_i\dot{x}^i-L_0~=~L_0, \qquad p_i~:=~\frac{\partial L_0}{\partial \dot{x}^i}~=~2g_{ij}(x) \dot{x}^j, $$ se conserva en la cáscara, es decir $$\tag{5}\dot{L}_0~\approx~ 0.$$ [Este argumento (5) no no funcionan para la acción de reparametrización-invariante (2), donde la función de energía $h=0$ desaparece de forma idéntica].

  10. (Supongamos para simplificar que $L_0$ no es cero en la cáscara, y dejar el caso en el que $L_0$ es cero en la cáscara como un ejercicio para el lector). Debido a la ec. (5), una solución de las ecuaciones EL para $L_0$ es también una solución a las ecuaciones EL para $\sqrt{L_0}$ : $$\tag{6} \frac{d}{d\lambda}\frac{\partial \sqrt{L_0}}{\partial \dot{x}^i}~\stackrel{(4)}{=}~\frac{d}{d\lambda}\frac{p_i}{2\sqrt{L_0}}~\stackrel{(5)}{\approx}~\frac{\dot{p}_i}{2\sqrt{L_0}}~\stackrel{\text{EL eq.}}{\approx}~\frac{1}{2\sqrt{L_0}}\frac{\partial L_0}{\partial x^i}~=~\frac{\partial \sqrt{L_0}}{\partial x^i}.$$

  11. Por el contrario, una solución de las ecuaciones EL para $\sqrt{L_0}$ es no necesariamente una solución a las ecuaciones EL para $L_0$ . Sin embargo, si dotamos la solución de las ecuaciones EL para $\sqrt{L_0}$ con una parametrización afín, entonces podemos obtener la ec. (5), e invertir el argumento (6), de modo que también es una solución de las ec. EL para $L_0$ .

--

$^1$ En la geometría pseudo-riemanniana, con firma mixta, la situación es más complicada, cf. por ejemplo este Puesto de Phys.SE. Además, allí el argumento de la raíz cuadrada $\sqrt{L_0}$ puede volverse negativo.

$^2$ Terminología y notación: Las ecuaciones de movimiento (EOM) significan Ecuaciones de Euler-Lagrange (EL) . Las palabras en la cáscara y fuera de la cáscara se refieren a si se satisfacen o no los MOE. El $\approx$ significa igualdad módulo EOM.

0 votos

1. Hola, una aclaración rápida sobre el 10. anterior, donde se deja el caso $L_0=0$ como ejercicio. Se trataría de una línea de mundo nula, y siempre he pensado que una aproximación por principios variacionales no está bien definida en ese caso 2. A no ser que se haga algo curioso como introducir campos auxiliares (multiplicadores de Lagrange). ¿Es eso lo que tenías en mente aquí, o se puede hacer la aproximación de la ecuación E-L directamente para trayectorias nulas de alguna manera?

1 votos

Hola @GRrocks. Gracias por los comentarios. 1. La acción de raíz cuadrada no está bien definida, pero la acción de raíz no cuadrada sí. 2. Sí, un campo einbein es una posibilidad.

5voto

bechira Puntos 826

Después de pensar un poco en esto me doy cuenta de que la pregunta se debía a un error bastante tonto, la inversa no se mencionaba en las referencias habituales porque es trivial desde la prueba de la dirección habitual. Para futuras referencias incluiré aquí una versión rápida.

$$F = L^2/2$$ $$ EL[F] \equiv \frac{d}{ds}(\frac{\partial}{\partial \dot{x^\mu}}F)- \frac{\partial}{\partial x^\mu}F \\ = \frac{d}{ds}(\frac{\partial L}{\partial \dot{x^\mu}} L)- \frac{\partial L}{\partial x^\mu}L\\ =\left(\frac{d}{ds}\left(\frac{\partial L}{\partial \dot{x^\mu}}\right) -\frac{\partial L}{\partial x^\mu}\right)L+ \frac{\partial L}{\partial \dot{x^\mu}} \frac{dL}{ds}\\ $$

Para la ecuación geodésica (de interés aquí) siempre es posible elegir la parametrización donde $\frac{dL}{ds}=0$ en tales parametrizaciones el último término desaparece y tenemos:

$$EL[F] = EL [L]$$ Por lo tanto, las ecuaciones de movimiento resultantes son equivalentes.

3voto

LiewChuan Puntos 11

Creo que hay algún tipo de error cerca del final de la primera respuesta (joshphysics), posiblemente en el paso variacional que implica la integración por partes. A no ser que se hayan utilizado algunas suposiciones más. De lo contrario, la conclusión implícita parece ser que la eqns EL para cualquier funcional con integrando $F(x(s),\dot{x}(s))$ son generalmente las mismas que para el funcional con integrando $F^2$ .

[Un contraejemplo sencillo es $F = x(s)$ Aunque me doy cuenta de que esto se aleja de la geodésica. Entonces la ecuación EL para $F^2$ sería $x=0$ mientras que la ecuación EL para $F$ mismo sería $1=0$ .]

De todos modos, volviendo a la pregunta original, a menudo las palabras " Cauchy-Schwarz " aparecen al relacionar $\int f^2$ a $\int f$ .

Ignorando eso, y mirando sólo las ecuaciones EL, considere la minimización/maximización de las dos funcionales $\int F^2 ds$ y $\int F ds$ (con el mismo parámetro y bcs), donde $F = F(x(s),\dot{x}(s))$ . ¿Cuándo son equivalentes estos dos problemas (o al menos, uno implica al otro)?

Las ecuaciones EL para los dos casos son generalmente diferentes (véase el contraejemplo anterior), pero una condición para la equivalencia es $dF/ds = 0$ (cuando se evalúa en una solución de cualquiera de las dos ecuaciones EL).

También tenemos las identidades de Beltrami para cada caso, que son primeras integrales de las ecuaciones EL (véase la conservación de la energía): para $F$ tenemos $\dot{x}_i\frac{\partial F}{\partial \dot{x}_i} - F = c_1$ y para $F^2$ tenemos $\dot{x}_i\frac{\partial (F^2)}{\partial \dot{x}_i} - F^2 = c_2$ (*).

Restringiendo al problema de las geodésicas en alguna superficie, tenemos un integrando de la forma $F = \sqrt{g_{ij}(x)\dot{x}^i\dot{x}^j}$ y esto satisface $\dot{x}_i \frac{\partial F}{\partial \dot{x}_i} = F$ (idénticamente), y también satisface $\dot{x}_i \frac{\partial (F^2)}{\partial \dot{x}_i} = 2F^2$ (+).

Supongamos que tenemos una solución a la ecuación EL para $F^2$ y, por tanto, a la identidad Beltrami $(*)$ para $F^2$ . Entonces (*) y (+) juntos dan $F^2 = c_2$ es decir $F$ es constante cuando se evalúa en nuestra solución.
[esto es esencialmente el punto 9 de la respuesta de Qmechanics] Por lo tanto, la condición de equivalencia mencionada anteriormente ( $dF/ds = 0$ ) se satisface, por lo que la ecuación EL para $F$ también se satisface.

Lo contrario no es válido en general, ya que la identidad de Beltrami para tal $F$ se satisface de forma idéntica (con $c_1 = 0$ ), por lo que no aporta nada "extra".

0 votos

Tienes razón, he cometido un error. Me alegro de que lo hayas detectado. Resulta que, de hecho, descuidé cierto término que contenía $dL/ds$ en la integración por partes, y encuentro, exactamente como indicas, que los problemas de EL son equivalentes siempre que $dL/ds = 0$ . Gracias por la atenta lectura lo editaré pronto.

i-Ciencias.com

I-Ciencias es una comunidad de estudiantes y amantes de la ciencia en la que puedes resolver tus problemas y dudas.
Puedes consultar las preguntas de otros usuarios, hacer tus propias preguntas o resolver las de los demás.

Powered by:

X